The Fibonacci numbers $F_0, F_1, F_2, . . .$ are defined inductively by $F_0=0, F_1=1$, and $F_{n+1}=F_n+F_{n-1}$ for $n \ge 1$. Given an integer $n \ge 2$, determine the smallest size of a set $S$ of integers such that for every $k=2, 3, . . . , n$ there exist some $x, y \in S$ such that $x-y=F_k$. Proposed by Croatia
Problem
Source: ISL 2020 C4
Tags: Fibonacci, combinatorics, IMO Shortlist, IMO Shortlist 2020, additive representation
21.07.2021 02:13
21.07.2021 02:18
The answer is $\left \lceil \frac{n}{2} \right \rceil + 1$, which is achievable by the following construction: \[ \{ F_0, F_2, \dots, F_{2 \cdot \lceil n/2 \rceil} \} \]Now, to prove the bound, construct a graph $G$ with elements of $S$ as its vertices, and for each $1 \le k \le \lceil n/2 \rceil$, connect two pair of unique vertices $(x,y)$ with an edge if $x - y = F_{2k - 1}$, using the fact that $F_1 = F_2$ and the condition of the problem. Claim. $G$ has no cycle. Proof. Suppose otherwise, that it contains a cycle with elements $(x_1, x_2, \dots, x_{m})$. Consider the largest difference in this cycle (WLOG it is $(x_1,x_m)$ with $|x_1 - x_m| = F_{2i + 1}$.) Therefore, by assumption, since $(x_1, x_2), (x_2, x_3), \dots, (x_{m - 1}, x_m)$ are all an edge, then $|x_{j + 1} - x_j|$ are the elements of $\{ F_1, F_3, \dots, F_{2i - 1} \}$ and are all distinct. However, notice that by Triangle Inequality, \begin{align*} F_{2i + 1} &= |x_{m} - x_1| \\ &\le \sum_{j = 1}^{m - 1} |x_{j + 1} - x_j| \\ &\le F_1 + F_3 + \dots + F_{2i - 1} \\ &= F_{2i} \end{align*}from which this gives us a contradiction. Therefore, this graph $G$ is a tree; and since $G$ needs to have $\lceil n/2 \rceil $ edge, then it must have at least $\lceil n/2 \rceil + 1$ vertices.
21.07.2021 02:34
Lemma: $\forall n\geq 2,\ F_n>\sum_{i=1}^{\lceil\frac{n-2}2\rceil}F_{n-2i}$. proof: induction on $n$. For the base case $n=2$ and $n=3$, $F_2, F_3>0$ is trivial. Assume that for $n=k-2$, the lemma holds. For $n=k$, $F_k=F_{k-1}+F_{k-2}>2F_{k-2}>F_{k-2}+\sum_{i=2}^{\lceil\frac{k-2}2\rceil}F_{k-2i}=\sum_{i=1}^{\lceil\frac{k-2}2\rceil}F_{k-2i}$. $\therefore$ by induction, the lemma holds for all $n\geq 2$. Let $S_n$ denote the set $S$ with the smallest size that satisfies the given condition. Let's have an induction on $n$ to prove that $|S_n|\geq\lceil\frac{n+2}2\rceil$. For the base case $n=2$ and $n=3$, $|S_2|\geq 2$ and $|S_3|\geq 3$ is trivial. Assume that for $n=k-2$, $|S_{k-2}|\geq\lceil\frac k2\rceil$. For $n=k$, if $|S_{k-2}|\geq\lceil\frac{k+2}2\rceil$ then by $|S_k|\geq |S_{k-2}|$ we are done. Otherwise, $|S_{k-2}|=\lceil\frac k2\rceil$. Construct a graph $G$, where $V(G)=S_{k-2}$, and edge $ab$ exists $\iff\exists 1\leq i\leq \lceil\frac k2\rceil-1$ s.t. $|a-b|=F_{k-2i}$. By lemma, there is no cycle in this graph; by the assumption of the induction, edge $ab$ with $|a-b|=F_{k-2i}$ exists $\forall 1\leq i\leq \lceil\frac k2\rceil-1$, and there are at least $\lceil\frac k2\rceil-1$ edges in total. $\therefore G$ is a tree, and the max element $-$ the min element of $S_{k-2}\leq\sum_{i=1}^{\lceil\frac k2\rceil-1}F_i<F_k$ $\therefore |S_k|>|S_{k-2}|$ $\therefore |S_k|\geq\lceil\frac{k+2}2\rceil$ $\therefore$ by induction, $\forall n\geq 2,\ |S_n|\geq\lceil\frac{n+2}2\rceil$ Construction of $S_n$: $\{F_{2i}|0\leq i\leq\lceil\frac n2\rceil\}$.
21.07.2021 02:44
The answer is \(k=\lceil\frac n2\rceil+1\), achieved by \(S=\{F_0,F_2,F_4,\ldots,F_{2k-2}\}\). I will show for each \(k\ge2\), if \(|S|=k\), then \(S-S\) contains at most \(2k-3\) distinct Fibonacci numbers. (If \(k=1\), then at most 0.) To this end, we strong induct on \(k\), with base cases \(k=1\) and \(k=2\) trivial. Draw a graph \(G\) with \(k\) vertices representing the \(k\) elements of \(S\). For each (distinct) Fibonacci number \(f\), draw an edge between one pair of nodes \((u,v)\) with \(|u-v|=f\). (If multiple \((u,v)\) exist, choose one.) It is equivalent to show at most \(2k-3\) edges are drawn for \(k\ge2\) and none for \(k=1\). Let \(F_t\) be the longest edge drawn. I contend that upon deleting the edge \(F_t\) and the edge \(F_{t-1}\) (if it exists), the graph is disconnected. Indeed, if a path exists between the two original endpoints of the \(F_t\) edge, then the sum of the lengths of the edges in this path is at most \[\sum\text{edge length}\le F_2+F_3+\cdots+F_{t-2}<F_t,\]contradiction. Now split \(G\) into \(G_1\sqcup G_2\) with \(k_1\) and \(k_2\) vertices. Since \(k\ge3\) we assume without loss of generality \(k_1\ge2\). Then \(G_1\) has at most \(2k_1-3\) edges by inductive hypothesis, and \(G_2\) has at most \(2k_2-2\) edges (taking the case \(k_2=1\) into consideration), so the number of edges in \(G\) is \[\#\text{edges in }G\le2+(2k_1-3)+(2k_2-2)=2k-3.\]
22.07.2021 08:38
\[ \begin{tabular}{p{12cm}} The answer is $\boxed{\left\lceil \dfrac{n}{2} \right\rceil+1}$. Bound and construction will be established later. \end{tabular}\]An inequality problem which appears like an induction-combi on the surface. That said, the ineq (finally) worked after going in circles to chase the right induction. $\color{green} \rule{5.8cm}{2pt}$ $\color{green} \clubsuit$ $ \boxed{\textbf{An Ineq Everyone Knows.}}$ $\color{green} \clubsuit$ $\color{green} \rule{5.8cm}{2pt}$ This whole proof hangs on these two statements: 1. $F_k > F_{k-2}+F_{k-3}+\ldots+F_2$ (more specifically, the LHS and RHS differ by $2$); 2. $F_2+F_3+F_5+\ldots+F_{2k-1} = F_{2k}$. $\color{green} \rule{25cm}{0.3pt}$ The $\boxed{\textbf{Proof}}$ is relatively simple. Note that by $F_{a+1} = F_a+F_{a-1}$ applied to $a = k$ and $a = 2k+1$ respectively implies the induction step on $n = k$ to $n = k+1$. $\color{green} \rule{25cm}{0.3pt}$ We now consider the elements of $S$ to be $s_1 < s_2 < \ldots < s_{k+1}$, and let $d_i = s_{i+1} - s_i$ for $1 \leq i \leq k$ (so there are exactly $k$ $\textsf{consecutive}$ differences). This will imply that for any $2 \leq k \leq n$, $F_k$ is $\textit{representable}$ as $d_i+d_{i+1}+\ldots+d_j$ for some $1 \leq i \leq j \leq n$. $\color{green} \rule{25cm}{0.3pt}$ We will first prove that $n \leq 2k$, and there exists a construction for each $n \leq 2k$. If this is true, then $|S|-1 = k \leq \dfrac{n}{2}$; and for $|S| = k+1 \leq \dfrac{n}{2}+1$, there exists a configuration where $F_2,F_3,\ldots,F_n$ all show up. Then, for each $d_i$, let its $\color{green} \textit{\textbf{ID}}$ to be the smallest $D_i$ so that $F_{D_i}$ $\color{green} \textit{\textbf{passes through}}$ $d_i$. That is, $F_{D_i}$'s representation above contains $d_i$. If no such number exists, let $D_i = 0$.
$\color{magenta} \rule{6.2cm}{2pt}$ $\color{magenta} \clubsuit$ $\color{red} \boxed{\textbf{A Bold Claim on Size Only.}}$ $\color{magenta} \clubsuit$ $\color{magenta} \rule{6.2cm}{2pt}$ If $\{R_1,R_2,\ldots,R_l\} = \{D_1,D_2,\ldots,D_k\} - \{0\}$, where $R_1 < R_2 < \ldots < R_l$, then $R_2 \leq R_1 + 1$ and $R_{a+1} \leq R_a+2$ for $2 \leq a \leq l-1$. $\color{red} \rule{25cm}{0.3pt}$ $\color{red} \spadesuit$ $\color{red} \boxed{\textbf{Proof.}}$ $\color{red} \spadesuit$ Let there exists an index $a$ so that $R_{a+1} \geq R_a+3$. Consider the $d_i$s with ID at most $R_a$ as follows:
So, $F_{R_a+1}$ and $F_{R_a+2}$ must only pass through/cover those $d_i$s with ID at most $R_a$. In establishing the above bound, we will discard and throw $F_{R_a+1}$ outside the equation; we'll only establish that $F_{R_a+2}$ is too big a number to fit only within those $d_i$s (hence implying that some other number will have ID $R_{a}+2$, at most). Now consider the sum of $F_2$ up to $F_{R_a}$ in their $\color{red} \textit{\textbf{representations}}$; this implies that a sum of $F_2 + F_3 + \ldots + F_{R_a}$ includes one (if not more) addition of each $d_{i_1},d_{i_2},\ldots,d_{i_m}$ since each of them must be covered at least once. However, if $F_{R_a+2}$ is only exclusive to them, then their total will be $\textbf{at least}$ $F_{R_a+2}$, which is more than $F_2 + F_3 + \ldots + F_{R_a}$, an addition of each of them (if not more). So, we have a contradiction. $\blacksquare$ $\blacksquare$ Note that here we rely on $F_{R_a+2}$'s superiority, established at $\color{green} \clubsuit$ $ \boxed{\textbf{An Ineq Everyone Knows}}$ $\color{green} \clubsuit$. $\color{cyan} \rule{10.5cm}{2pt}$ $\color{cyan} \clubsuit$ $ \boxed{\textbf{Semi-Finishing: Mechanically Repeating the Process.}}$ $\color{cyan} \clubsuit$ $\color{cyan} \rule{10.5cm}{2pt}$ We claim that $\max_{i=1}^k(D_i) \leq 2k-1$, and $n \leq 2k$. $\color{cyan} \rule{25cm}{0.3pt}$ The first one is straightforward: \[ \max_{i=1}^k(D_i) = R_l \leq \dots \leq R_2+(2l-4) \leq R_1+(2l-3) = 2l-1 \leq 2k-1 \]as $l \leq k$. The second one is done by summing similarly to $\color{red} \clubsuit$ $\color{red} \boxed{\textbf{A Bold Claim on Size Only}}$ $\color{red} \clubsuit$: \begin{align*} \sum_{\delta = 1}^k d_{\delta} &\leq F_2+F_3+\ldots+F_{R_l} \\ &\leq F_2+F_3+\ldots+F_{2k-1} \\ &< F_{2k+1} \end{align*}which implies that $F_{2k+1}$ cannot exist given that there are only $k$ $d_i$s. $\color{blue} \rule{4.8cm}{2pt}$ $\color{blue} \diamondsuit$ $\color{blue} \boxed{\textbf{Construction Finish.}}$ $\color{blue} \diamondsuit$ $\color{blue} \rule{4.8cm}{2pt}$ We will prove that we can form $F_2$ up to $F_{2k}$ by only using $k$ $d_i$s. Set $d_1 = F_2$, $d_2 = F_3$, $d_3 = F_5$, and so on, until $d_k = F_{2k-1}$. Here, we can obtain the values of $F_{2i}$, $2 \leq i \leq k$ by letting \[ s_{i+1}-s_1 = F_2+F_3+\ldots+F_{2i-1} = F_{2i} \]by $\color{green} \clubsuit$ $ \boxed{\textbf{An Ineq Everyone Knows}}$ $\color{green} \clubsuit$. We are done. $\blacksquare$ $\blacksquare$ $\blacksquare$
25.07.2021 09:59
24.08.2021 15:49
MathLuis wrote: The Fibonacci numbers $F_0, F_1, F_2, . . .$ are defined inductively by $F_0=0, F_1=1$, and $F_{n+1}=F_n+F_{n-1}$ for $n \ge 1$. Given an integer $n \ge 2$, determine the smallest size of a set $S$ of integers such that for every $k=2, 3, . . . , n$ there exist some $x, y \in S$ such that $x-y=F_k$. $Proposed \; by \; Croatia$ Killed when you realize the answer. We claim that the answer is $|S| \geq \lceil \frac{n}{2} \rceil + 1$. For a given natural number $n \geq 2$, consider $S = \{ 0, F_0, F_2, \dots F_{2 \lceil \frac{n}{2} \rceil }$. We see that $F_{2i} = F_{2i} - 0$ and $F_{2i+1} = F_{2i+2} - F_{2i}$ according to the recursion. To prove that $|S| = k \geq \lceil \frac{n}{2} \rceil + 1$, we draw a graph $G$ whose vertices are the distinct elements in $S$, such that the vertices $v_1, v_2 \dots v_k$ of $G$ represent elements $s_1, s_2 \dots s_k$ of $G$. Draw an edge between two vertices $v_i, v_j$ if the pair $(i, j)$ minimizes $i + j$ while $|s_i - s_j| = F_t$ for some odd number $t \in 2, 3, \dots $. We observe that the graph cannot have a cycle since $F_{2t+1} \geq \sum\limits_{i=0}^{t-1} F_{2i+1} = F_{2t}$ and this means that $G$ is a forest .This means that $V(G) = E(G) + c(G)$, but we see that $E(G) = \left \lceil \frac{n}{2} \right \rceil$ and this means that $|S| \geq V(G) = E(G) + c(G) \geq \left \lceil \frac{n}{2} \right \rceil + 1$ as desired.
22.04.2022 14:22
02.05.2022 22:50
The answer is $\left\lceil\frac{n}{2}\right\rceil+1$. To see that it's sufficient, consider the set $\{F_0,F_2\ldots\}$. To see that it's necessary, consider the graph formed by elements of $S$ where any two elements differing by a fibonacci number are connected by an edge. Now, color an edge red if it connects two number differing by $F_{2m}$ and blue if it connects two numbers differing by $F_{2m-1}$ for positive integer $m$. Note that there can be no red cycle and no blue cycle by considering the longest edge in each cycle. Hence, there are at most $|S|-1$ blue edges and $|S|-1$ red edges, but notice however that the necessary edge with length $F_2=F_1$ is colored twice, so there can be at most $2|S|-3$ edges in total in a satisfactory $S$, which finishes.
17.02.2023 08:03
The answer is $\lfloor \tfrac{n}2 \rfloor + 1$ for which we provide the following construction: \[S_n=\{0\} \cup \{F_i \mid i\equiv n\pmod 2, i>0\}\]which trivially works. Now, to prove the bound, we need to show that we cannot make $F_2$, $F_3$, $\dots$, $F_{2k-1}$ all with differences of a $k$-sized set. For this, note that \begin{align*} F_{2m} &> F_{2m-1}+F_{2m-3}+\dots + F_{3} \\ F_{2m-1} &> F_{2m-2} + F_{2m-4}+\dots + F_2 \end{align*}We proceed by contradiction: construct a $k$-vertex graph with each vertex as a member of the set. For each $F_2$, $F_3$, $\dots$, $F_{2k-1}$, draw exactly one red edge between two vertices whose corresponding elements in the set differ by that fibonacci number with even index, and one blue edge for a fibonacci number with odd index. Note that $2k-2$ edges are drawn, $k-1$ of color red and $k-1$ of color blue. By the two inequalities given above, there are no cycles. Thus, the red and blue vertices each for a tree. Now to finish, consider the edge corresponding with $F_{2k-1}.$ We know from the same inequalities that had us conclude the lack of cycles, that were this edge colored red, then there still may not be a cycle. This is simply not possible. We are done.
06.09.2023 02:53
The answer is $\lceil\tfrac n2\rceil+1$, achievable by taking $S=\{F_0,F_2,F_4,\ldots,F_{2k-2}\}$. Construct a graph $G$ on $S$, and for each $j \in \{F_3, F_5, \ldots, F_{2k-1}\}$, draw an edge between a single pair $(v, w)$ of vertices such that $|v-w|=j$. Furthermore, label each edge $v \sim w$ with $|v-w|$. Claim: $G$ has no cycles. Proof. Assume for contradiction that $G$ has a cycle $\mathcal{C}$. Let the largest label in $\mathcal{C}$ be $F_{2m+1}$. By the Triangle inequality, the sum of all other labels in the cycle is at least $F_{2m+1}$, however this sum is at most \[ \sum_{i=1}^m F_{2i-1} = F_{2m},\]which is a contradiction. Thus $G$ must be a tree, so it has at least $\lceil\tfrac n2\rceil+1$ vertices, and we are done.
12.04.2024 15:28
My solution is similar to the ones posted on this thread. We claim that the smallest possible size of $S$ is $\left\lceil\frac n2\right\rceil+1$. The set $S=\left\{F_0, F_2, \ldots, F_{2\left\lceil\frac n2\right\rceil}\right\}$ works as a construction. The main idea is that $F_1+F_3+\cdots+F_{2t-1}=F_{2t}$. This can be shown induction, note that $t=1$ works, and \[F_{2t+2}=F_{2t+1}+F_{2t}=F_{2t+1}+F_{2t-1}+\cdots+F_3+F_1,\]so the summation is correct. Suppose there exists a set $S$ of $m$ elements with $1\leq m\leq \left\lfloor\frac n2\right\rfloor$ that satisfies the given condition. Consider a graph $G$ whose vertices represents the elements in $S$. Then, for every $1\leq k\leq \frac{n+1}2$, vertices $x,y$ of $G$ are adjacent if $x-y=F_{2k-1}$ and to ensure uniqueness, take the smallest such pair. There are exactly $\left\lfloor\frac{n+1}{2}\right\rfloor$ edges in $G$. Claim: $G$ has no cycles. Proof. Suppose there is some cycle of elements $x_1, x_2, \ldots, x_c, x_1$ numbered such that $F_{2d-1}=|x_c-x_1|>|x_i-x_{i-1}|$ for every $2\leq i\leq c$. Then, $d\geq c$ is forced. Then, by Triangle Inequality of Modulus, we have \[F_{2d-1}=|x_c-x_1|\leq |x_2-x_1|+|x_3-x_2|+\cdots+|x_c-x_{c-1}|\leq F_1+F_3+\cdots+F_{2c-1}=F_{2c}\]which is impossible as $d\geq c$. Hence, $G$ has no cycles. $\blacksquare$ From our claim, we have that $G$ is a (possibly disconnected) tree. Then, $1+\left\lceil\frac {n}2\right\rceil=\left\lfloor\frac {n+1}2\right\rfloor=\#(\text{edges}) \leq \#(\text{vertices})-1=m-1$, which means $m\geq2+\left\lceil\frac n2\right\rceil$, giving us the required contradiction. $\blacksquare$